How Can We Find All Integer Pairs (k, n) That Satisfy This Factorial Equation?

  • MHB
  • Thread starter anemone
  • Start date
In summary, POTW #473 is a weekly problem on Brilliant.org that challenges users to solve complex math and science problems. The topic of this specific problem is solving for pairs of positive integers (k,n) in a factorial equation. A factorial equation is an equation in the form of n! = k, where n is a positive integer and k is the product of all positive integers from 1 to n. To solve for these pairs, one can use methods such as trial and error, substitution, or algebraic manipulation. The significance of this type of problem is that it can help in identifying patterns and relationships between numbers, and can be applied in various mathematical and scientific contexts.
  • #1
anemone
Gold Member
MHB
POTW Director
3,883
115
Here is this week's POTW:

-----

Find all pairs of positive integers $(k,\,n)$ such that $k!=(2^n-1)(2^n-2)(2^n-4)\cdots(2^n-2^{n-1})$.

-----

 
Physics news on Phys.org
  • #2
anemone said:
Here is this week's POTW:

-----

Find all pairs of positive integers $(k,\,n)$ such

-----
LHS= ##k! >(\frac ke)^k\sqrt{2\pi k}\frac 1{e^{12k+1}}##
RHS##=(2^n-1)(2^n-2)(2^n-4)\cdots(2^n-2^{n-1})##
##=(2^n)^n\Pi_1^n(1-2^{-r})##
##=2^{n^2}e^{\Sigma_1^n\ln(1-2^{-r})}##
##<2^{n^2}e^{\Sigma_1^n-2^{-r}}##
##=2^{n^2}e^{-1}##
So ##(\frac ke)^k\sqrt{2\pi k}\frac 1{e^{12k+1}}<2^{n^2}e^{-1}##
##(\frac ke)^k<2^{n^2}##
Define ##d_2(m)## as the number of times 2 divides m.
##\frac 12k<d_2(LHS)<k##
##d_2(RHS)=\frac 12n(n-1)##
Hence ##k<n(n-1)<2k##
If ##n\geq N## then ##n^2\frac{N-1}N<n(n-1)<2k##.

##(\frac ke)^k<2^{2k\frac N{N-1}}##
##k<2^{2\frac N{N-1}}e##
With N=5 this yields
##k\leq 15##
##n<6##
Thus, if ##n\geq 5, n\leq 5##, hence ##n\leq 5##.

I haven't been through all the possibilities yet.. no time now … but the only two solutions that are obvious are n=1, k=1; n=2, k=3.
 
Last edited:
  • Like
Likes anemone
  • #3
Here's an informal argument:

Let $$a(n) =(2^n-1)(2^n-2)(2^n-4)\cdots(2^n-2^{n-1})$$And note that$$a(n) = (2^n-1)(2^{n-1})a(n-1)$$If we let ##t(n)## be the powers of ##2## in the prime factorisation for ##a(n)## then we see that:$$t(n) = \frac{n(n-1)}{2}$$And note that ##t(n)## increases rapidly with ##n##.

Now we look at the powers of ##2## in ##k!## and we note that this increases much less rapidly.

In any case, we can look for the factorials that have the same power of ##2## for the relevant ##n##. E.g:
$$n = 5, t(n) = 10, k = 12 \ \text{or} \ 13$$We can check that ##a(n) \ne k!## in these cases. The next cases are:
$$n = 6, t(n) = 15, k = 16 \ \text{or} \ 17$$$$n = 7, t(n) = 21, k = 24 \ \text{or} \ 25$$And for this value of ##n## the possible values of ##k!## are already much greater than ##a(n)##. If we keep going:
$$n = 8, t(n) = 28$$ there are no possible values of ##k##
$$n = 9, t(n) = 36$$ there are also no possible values of ##k##. Moreover, for the nearest values of ##k## we see that ##k!## is now much larger than ##a(n)## it's clear that there can be more solutions.

You could look more formally at how ##a(n)## and the possible values of ##k## are increasing to formalise the argument somewhat.
 
Last edited:
  • Like
Likes anemone
  • #4
I only came up with this after reading the other solutions, so it's not very original but I think hits the heart of the problem.

for ##0<j<2^{n-1}##, ##j## and ##2^{n-1}+j## have the same number of factors of ##2## - if ##2^a|j## then ##j<n-1## so ##2^a | 2^{n-1}##, and hence ##j = j+2^{n-1}## mod ##2^a##

Matching ##2^{n-1}+1## with ##1## all the way through ##2^{n}-1## with ##2^{n-1}-1##, we see the product in the question has the same number of 2s as ##2^{n-1}!##This doesn't agree with perok's numbers, for ##n=7## this gives ##k=64##, but wolfram alpha agrees these both are divisible by ##2^{63}##

https://www.wolframalpha.com/input?i=factor+(2^6)!

https://www.wolframalpha.com/input?i=factor+(2^7-1)!/(2^6-1)!

So ##k## must be ##2^{n-1}## or ##2^{n-1}+1##. For ##k=1## these products are the same. For ##k=3## you also get equality. In general in our mapping of numbers in the ##(2^n-1)!/(2^{n-1}-1)!## to the numbers in ##2^{n-1}!## to establish the power of 2 correpondence, each number in the former was at least twice as large as the latter except for the ##2^{n-1}## which we didn't map to a smaller number since it's in both products, so to get equality you need ##k=1##, or you need##k=2^{n-1}+1## to get an extra factor to try to catch up. But even then we just get an extra factor of ##2^{n-1}+1## when we're trying to compete with ##2^{n-1}-1## extra powers of two (so a factor of 2 raised to that quantity). So we need ##n-1 \geq 2^{n-1}-1## which is only true for ##n=1## or ##n=2##.
 
Last edited:
  • #5
Here's a simpler informal argument:

Let $$a(n) =(2^n-1)(2^n-2)(2^n-4)\cdots(2^n-2^{n-1})$$And note that$$a(n) = (2^n-1)(2^{n-1})a(n-1)$$We can now analyse the prime factors of ##a(n)## recursively by looking at the sequence ##2^n-1##:
$$3, 7, 15, 31, 63, 127 \dots$$Immediately, we see that prime numbers like ##31## and ##127## appear early in the factorisations. But, don't appear in ##k!## until ##k =31## or ##k = 127## respectively.

It's clear (although not so easy to prove formally) that this sequence can never produce another factorial for larger ##n##. It has far too many powers of ##2## compared to the other primes and has larger primes appearing too early in the sequence.
 

Related to How Can We Find All Integer Pairs (k, n) That Satisfy This Factorial Equation?

1. What is the factorial equation being solved in POTW #473?

The factorial equation being solved is k! = n! + 4.

2. How many solutions are there for the equation in POTW #473?

There are infinitely many solutions for this equation.

3. Can you explain the process for solving this factorial equation?

The first step is to simplify the equation by dividing both sides by n!. This will give us k(k-1)(k-2)...(k-n+1) = 4. Then, we can use trial and error or algebraic methods to find values of k and n that satisfy the equation.

4. Are there any restrictions on the values of k and n in this equation?

Yes, k and n must be positive integers. Additionally, n must be less than or equal to k.

5. How can this factorial equation be applied in real-world situations?

This equation can be used to solve problems involving combinations and arrangements, such as finding the number of ways to arrange a certain number of objects. It can also be used in probability calculations.

Similar threads

  • Math POTW for Secondary and High School Students
Replies
2
Views
788
  • Math POTW for Secondary and High School Students
Replies
1
Views
1K
  • Math POTW for Secondary and High School Students
Replies
1
Views
921
  • Math POTW for Secondary and High School Students
Replies
1
Views
1K
  • Math POTW for Secondary and High School Students
Replies
1
Views
837
  • Math POTW for Secondary and High School Students
Replies
1
Views
844
  • Math POTW for Secondary and High School Students
Replies
14
Views
1K
  • Math POTW for Secondary and High School Students
Replies
1
Views
826
  • Math POTW for Secondary and High School Students
Replies
1
Views
824
  • Math POTW for Secondary and High School Students
Replies
1
Views
974
Back
Top